subject
Mathematics, 18.03.2021 14:00 abhibhambhani

Solve the following using GEMDAS and PEMDAS rule. Write your solution. 1.) 5 (3±9equal2) - 7 ( 14 ) ± 30 (-15)×2
2) ( 10±3 ) ÷ 3 ± 13 (-30)
3.) 4±9×8÷2 (1±2³)
4.) 13 ± 30 ± 2²

ansver
Answers: 2

Another question on Mathematics

question
Mathematics, 21.06.2019 17:20
What other information do you need to prove triangle dac=bca by asa
Answers: 1
question
Mathematics, 21.06.2019 17:40
Find the value of x that will make l ll m
Answers: 2
question
Mathematics, 21.06.2019 18:00
When the ball has traveled a distance of
Answers: 1
question
Mathematics, 21.06.2019 23:00
Spencer has 1/3 pound of nuts he divides them equally into 4 bags what fraction of a pound of nuts is in each bag
Answers: 1
You know the right answer?
Solve the following using GEMDAS and PEMDAS rule. Write your solution. 1.) 5 (3±9equal2) - 7 ( 14 )...
Questions
question
Mathematics, 18.10.2020 15:01
question
Social Studies, 18.10.2020 15:01
question
Social Studies, 18.10.2020 15:01
question
Mathematics, 18.10.2020 15:01
question
Mathematics, 18.10.2020 15:01
Questions on the website: 13722367